LSAT and Law School Admissions Forum

Get expert LSAT preparation and law school admissions advice from PowerScore Test Preparation.

User avatar
 Dave Killoran
PowerScore Staff
  • PowerScore Staff
  • Posts: 5850
  • Joined: Mar 25, 2011
|
#27067
Complete Question Explanation
(The complete setup for this game can be found here: lsat/viewtopic.php?t=8627)

The correct answer choice is (E)

Another List question, this time a Local question with the stipulation that W is selected. Do not make the mistake of thinking that because W is reduced that G and S are both reduced! This is a mistaken reversal of the rule. Answer choice (A) is incorrect since two of L, M, and R must be reduced and only M is reduced. Answer choice (B) is incorrect since both N and R are reduced, or alternately, because all three of L, M, and R are reduced. Answer choice (C) is incorrect because both P and L are reduced. Answer choice (D) is incorrect since both N and S are reduced. Consequently, answer choice (E) is correct, and we now know that the hypothetical W-M-P-R-S is a valid solution to the game.
User avatar
 Abhinusa
  • Posts: 2
  • Joined: Jun 08, 2021
|
#87682
is D also incorrect because of the LMR rule?
 Robert Carroll
PowerScore Staff
  • PowerScore Staff
  • Posts: 1787
  • Joined: Dec 06, 2013
|
#87754
Abhinusa,

Yep! That's another reason answer choice (D) is out.

Robert Carroll

Get the most out of your LSAT Prep Plus subscription.

Analyze and track your performance with our Testing and Analytics Package.